Printable Version of Topic

Click here to view this topic in its original format

Unmanned Spaceflight.com _ MSL _ Temperature and pressure at Gale

Posted by: Seryddwr Sep 30 2012, 03:23 PM

Just a quick query from someone with no background in science. Obviously, MSL has AFAIK not returned evidence of recent (i.e. years/decades) liquid water in its vicinity; however, I was interested by the following graphs:

08.21.2012: First Pressure Readings on Mars

http://mars.jpl.nasa.gov/msl/multimedia/images/?ImageID=4501

08.21.2012: Taking Mars' Temperature

http://mars.jpl.nasa.gov/msl/multimedia/images/?ImageID=4502

The first indicates that the pressure between 15 Aug and 18 Aug never dropped below c. 690 millibars; the second shows that, for a period of a couple of hours on 16 Aug, the temperature rose above freezing. If water had been present on the surface, then, would it have been liquid during this brief period? The pressure and temperature seemed to satisfy the conditions for liquid water as I understand them (indeed, the pressure seems to be high enough (just) on a 24-hour basis to allow for the presence of liquid water). Thanks in advance for your opinions (corroborative or not!) on this.

Posted by: ngunn Sep 30 2012, 03:33 PM

690 Pa = 6.9 mbar

Posted by: Seryddwr Sep 30 2012, 03:40 PM

6.9 - quite! 690 millibars would have been quite a discovery! ohmy.gif

Posted by: nprev Sep 30 2012, 03:51 PM

I doubt that the pressure on the surface ever exceeds 10 mb, and that would be at the bottom of Valles Marineris and some portions of the Hellas basin.

6.9 mb for Gale is probably about as good as it gets, plus or minus a few tenths...therefore, no possibility of sustaining liquid water.

Posted by: Eyesonmars Sep 30 2012, 04:03 PM

QUOTE (Seryddwr @ Sep 30 2012, 03:23 PM) *
If water had been present on the surface, then, would it have been liquid during this brief period? The pressure and temperature seemed to satisfy the conditions for liquid water as I understand them (indeed, the pressure seems to be high enough (just) on a 24-hour basis to allow for the presence of liquid water). Thanks in advance for your opinions (corroborative or not!) on this.

In theory the answer is yes. But the thinking is that the strong evaporative cooling at what is essentially a 0% relative humidity associated with these brief high temperatures would make a liquid state difficult to maintain or achieve. However, imagine placing a pan of water at ground level in sun versus shade (insulated from the ground) at the MSL site. The pan in full noonday sun would still evaporate but solar heating might offset evaporative cooling enough to keep it liquid while it evaporated. Not so in the shade. Note that our pan of water is assumed to have an initial temperature just above freezing because the boiling point on mars even at the bottom of Hellas will never be more than about 5-10c above freezing. This last fact makes it even more difficult to keep liquid water from icing over

Posted by: djellison Sep 30 2012, 04:31 PM

QUOTE (Eyesonmars @ Sep 30 2012, 09:03 AM) *
the boiling point on mars even at the bottom of Hellas will never be more than about 5c above freezing


This is the kicker. Are there conditions where liquid water could exist on the surface of Mars today. Yes. Two problems, where does it come from, and it would so very quickly evaporate away that its existence could only ever be transient.

Posted by: Eyesonmars Sep 30 2012, 04:50 PM

QUOTE (nprev @ Sep 30 2012, 03:51 PM) *
I doubt that the pressure on the surface ever exceeds 10 mb, and that would be at the bottom of Valles Marineris and some portions of the Hellas basin.

6.9 mb for Gale is probably about as good as it gets, plus or minus a few tenths...therefore, no possibility of sustaining liquid water.

The maximum surface pressure on mars occurs just after southern summer solstice. This is still many months away. At the Viking 2 site the average pressure at this time was near 10.2mb. The MSL site is another 2km or so LOWER than this. Using a scale height of 11km for mars suggest we might see pressures on the order of 11+ mb at this time compared to our current 7mb. Plugging in the numbers for Hellas (-8km) you can see that pressures can be as high as ~14mb.
For those interested in a good introduction to the Martian atmosphere (with Viking data ) ......
"The Surface of Mars" by Michael J. Carr (1980) Yale University Press
Still one of the best in my opinion

Posted by: Eyesonmars Sep 30 2012, 08:04 PM

QUOTE (djellison @ Sep 30 2012, 04:31 PM) *
.... and it would so very quickly evaporate away that its existence could only ever be transient.

An interesting ( at least for me) thought experiment is what would happen to a TALL glass of cold water at the bottom of Hellas under the mid day sun at southern summer solstice. Surprisingly the water would be stable for some time before evaporating ( or icing over). The trick is the small surface to volume ratio.

Posted by: djellison Sep 30 2012, 09:47 PM

It would boil, would it not?

Posted by: udolein Sep 30 2012, 11:06 PM

Liquid water is almost impossible under the current conditions:



The enclosed phase diagrams of water and carbon dioxid state clearly that the triple point of H2O is around 6 mbar and 0 deg Celsius, while for CO2 it is 5.1 bar (not mbar !) and -56.5 deg Celsius. This means: below 0 deg Celsius water is solid (ice) at 6 mbar and CO2 is a gas. At 7 deg Celsius as on sol 52 and 6.9 mbar air pressure water theoretically could be a liquid but it is most probably a gas. And CO2 is a gas always.

Cheers, Udo






Posted by: udolein Sep 30 2012, 11:19 PM

BTW: This site has the current weather readings: http://marsweather.com/data

Udo

Posted by: djellison Sep 30 2012, 11:25 PM

Quite- we''re dancing around a tiny tiny wedge at the low pressure end of the liquid part of the H2O phase diagram. Even with dramatic salt content, that end of the diagram doesn't change much.


Posted by: Eyesonmars Sep 30 2012, 11:37 PM

QUOTE (djellison @ Sep 30 2012, 09:47 PM) *
It would boil, would it not?

As shown in one of my previous post, the surface pressure may be as high as 14mb. At that latitude (40 south) on mars at that season mid day near surface air temperatures are well above freezing. So temperature and pressure are high enough. Rapid evaporative cooling could ice over our glass of ice water in the extreme dryness, even with air temps above freezing. But solar radiation might offset the evaporative cooling. If this happens, our glass of ice water might slowly warm a few degrees before it completely evaporates but because the boiling point might be as high as 10c at 14mb the water would evaporate before it ever warmed to the boiling point.
Like I said, it is a fun little thought experiment. Small changes in parameters drastically change the outcome. For instance, if the glass was sitting on black sand dunes (albedo .05) the infrared warmth ( dune temp could easily be 35-40c) might warm the ice water quickly enough for it to begin boiling before it completely evaporates

Posted by: serpens Sep 30 2012, 11:40 PM

Yep to djellison. And addressing reality rather than glasses of water on Mars, given the miniscule absolute humidity any overnight frost layer would be measured in microns. It would possibly sublimate while the temperature is below the freezing point and before it had a chance to change state to liquid.

Posted by: udolein Sep 30 2012, 11:44 PM

QUOTE (Eyesonmars @ Oct 1 2012, 01:37 AM) *
For instance, if the glass was sitting on black sand dunes (albedo .05) the infrared warmth ( dune temp could easily be 35-40c) might warm the ice water quickly enough for it to begin boiling before it completely evaporates

The phase transition under the conditions mentioned would happen directly from ice to gas. It evaporates. There would be no boiling at all. At 40c a liquid phase is impossible at 6-10 mbar air pressure.
In my above phase diagram the phase transition would happen from point D to F. No liquid phase at all.

Udo

Posted by: udolein Sep 30 2012, 11:59 PM

CO2 won't be a liquid at normal conditions due to the 5.1 bar triple point. This is the reason why dry ice sublimates directly to gaseous CO2. There would be no liquid phase in between. The phase transition would happen from D to F as well.

Udo

Posted by: Eyesonmars Oct 1 2012, 12:14 AM

QUOTE (udolein @ Sep 30 2012, 11:19 PM) *
BTW: This site has the current weather readings: http://marsweather.com/data

Udo

They also have ground temperatures, true ?
It would be nice to see ground temperatures also.
One reason, for instance, is that the temperature of the bottom, sides and other parts of the rover facing the surface and not in direct sunlight is largely in radiative equilibrium with the ground. Heat conduction via the atmosphere is negligible regardless of air temp.

Posted by: Eyesonmars Oct 1 2012, 12:33 AM

QUOTE (udolein @ Oct 1 2012, 12:44 AM) *
The phase transition under the conditions mentioned would happen directly from ice to gas. It evaporates. There would be no boiling at all. At 40c a liquid phase is impossible at 6-10 mbar air pressure.
In my above phase diagram the phase transition would happen from point D to F. No liquid phase at all.

Udo

Note that I am talking about magically placing a glass of ice water ( temp 1c or so) on the surface. Please see earlier post.
As I state, the 40c temp I mentioned is the temp of the sand, not the water. Radiative heating from the surrounding sand would probably be more important than conduction thru the bottom of our glass.

And I'm not clear on why you are referring to the phase diagram of CO2 in some of your post

Posted by: Eyesonmars Oct 1 2012, 01:36 AM

QUOTE (djellison @ Sep 30 2012, 11:25 PM) *
Quite- we''re dancing around a tiny tiny wedge at the low pressure end of the liquid part of the H2O phase diagram. Even with dramatic salt content, that end of the diagram doesn't change much.

Are you looking at a phase diagram designed for engineers ( most of them are as is the one udolien is using). If so the area of interest to atmospheric scientist will always be a "tiny tiny wedge". On one designed for Martian use the "tiny wedge" will occupy a whole page.

Posted by: nprev Oct 1 2012, 02:41 AM

Um. Let's put it this way, Eyes: We ain't gonna see any liquid water at Gale. We're a few billion years too late.

Let's deal with what is real.

Posted by: djellison Oct 1 2012, 04:13 AM

QUOTE (Eyesonmars @ Sep 30 2012, 05:36 PM) *
On one designed for Martian use the "tiny wedge" will occupy a whole page.


Yes it would - and on that page you would see that at these very low pressures, the temperature difference between melting and boiling is very small.

Posted by: Juramike Oct 1 2012, 02:01 PM

QUOTE (Eyesonmars @ Sep 30 2012, 07:33 PM) *
Note that I am talking about magically placing a glass of ice water ( temp 1c or so) on the surface.


Or take a glass of ice water and put it in a vacuum chamber and suddenly expose it to vacuum. [Done the equivalent lotsa times in a rotary evaporator].

Doug's right, it would boil. Bubbles of water vapor would form at imperfections in the glass of the surface and it would bloop up out of the glass and go everywhere. A combination of sudden cooling due to more evaporative surface area would make all the flung water droplets quickly turn to ice, which would then sublimate away.

[When this happens in the lab, it is due to the receiver trap bumping, and there is usually a "Dangit!" if any of the water drops land back in the previously dried compound].

(Water-->bloop--->freeze--------------------->sublimate)

The water-bloop-droplet-freeze sequence would happen on the order of a few seconds. If the glass has a rougher surface, it would foam and froth smoothly due to all the nucleation sites. If it was a really smooth surface, it would sit quietly for a second and then just release it in a massive spasm of boiling due to the fewer nucleation sites.

Posted by: ElkGroveDan Oct 1 2012, 02:13 PM

QUOTE (Eyesonmars @ Sep 30 2012, 06:36 PM) *
Are you looking at a phase diagram designed for engineers ( most of them are as is the one udolien is using). If so the area of interest to atmospheric scientist will always be a "tiny tiny wedge". On one designed for Martian use the "tiny wedge" will occupy a whole page.

I can make one the size of a highway billboard and it won't change the state of matter anywhere in the universe. As Doug has repeated several times you are talking about a miniscule portion of the phase diagram in every practical sense. You might as well obsess over how many angels could break-dance in that portion of the diagram without bumping into each other.

Posted by: Eyesonmars Oct 1 2012, 05:07 PM

Hey guys come on
Before you all pile on Please read my posts from the beginning.
I've never suggested that liquid water would be stable on mars in any realistic scenario.
I am addressing the question posed by the author of this thread.
In it I clearly state why liquid water is all but impossible on mars.
Then I offer some physical insight into my statement by offering a thought experiment.

I know it is so easy to take a single post out of context - who hasn't ?

(yet another Doug here)

Posted by: Eyesonmars Oct 1 2012, 05:20 PM

QUOTE (Juramike @ Oct 1 2012, 02:01 PM) *
Or take a glass of ice water and put it in a vacuum chamber and suddenly expose it to vacuum. [Done the equivalent lotsa times in a rotary evaporator].

Doug's right, it would boil. Bubbles of water vapor would form at imperfections in the glass of the surface and it would bloop up out of the glass and go everywhere. A combination of sudden coo anling due to more evaporative surface area would make all the flung water droplets quickly turn to ice, which would then sublimate away.

[When this happens in the lab, it is due to the receiver trap bumping, and there is usually a "Dangit!" if any of the water drops land back in the previously dried compound].

(Water-->bloop--->freeze--------------------->sublimate)

The water-bloop-droplet-freeze sequence would happen on the order of a few seconds. If the glass has a rougher surface, it would foam and froth smoothly due to all the nucleation sites. If it was a really smooth surface, it would sit quietly for a second and then just release it in a massive spasm of boiling due to the fewer nucleation sites.

Juramike,
I have followed your posts for years and have learned a great deal from you, especially the thought experiments you pose.

In your opinion what would happen to a glass of water with a temperature of 4c put into that chamber at a pressure of say 12mb. ? ignore other variables for the moment

Posted by: Eyesonmars Oct 1 2012, 05:46 PM

QUOTE (djellison @ Oct 1 2012, 04:13 AM) *
Yes it would - and on that page you would see that at these very low pressures, the temperature difference between melting and boiling is very small.

Yes
The temperatures and pressures we are used to dealing with on earth in engineering, meteorology, you name it,are rarely near the triple point of water. But on Mars we have an entire PLANET and atmosphere which is never very far from the triple point of H20. Small changes in conditions near the triple point can have major effects. I would imagine future residents of mars would see H20 behave in ways that would seem alien to us. Their very existence might depend on understanding how H20 behaves under Martian conditions.

Posted by: marsophile Oct 1 2012, 05:49 PM

I think one can make a case for transient wetting under certain conditions. Suppose there is overnight frost concentrated , let's say, in a cold trap area. In the early morning while the air temperature is still well below freezing, the thin frost cover might produce a green house effect on the soil beneath, especially for an east-facing slope, so that it warms above freezing. So there might be brief wetting underneath a vanishing frost cover.

Obviously this can only happen at locations and times where there is overnight frost. The Opportunity rover has shown this can occur even in equatorial regions. It would be nice to have orbital surveys of where and when frost occurs.

Posted by: Juramike Oct 1 2012, 06:53 PM

QUOTE (Eyesonmars @ Oct 1 2012, 12:20 PM) *
In your opinion what would happen to a glass of water with a temperature of 4c put into that chamber at a pressure of say 12mb. ? ignore other variables for the moment


(Assuming bp. of H2O at 12 mb is at 10 C)

It would probably behave similar to a low boiling liquid under terrestrial conditions. I'd use diethyl ether as an example (b.p. 35 C). It won't boil at a normal room temperature of 25 C, but it will evaporate very quickly.
As it evaporates, it will cool.

Water should do the same thing under vacuum, but water's melting point is pretty high, so it will evaporatively cool and then freeze. After it is frozen it will sublimate. Some wierdness might occur if the glass of water is deep enough, the water would freeze on top and maybe you'd get liquid water sealed up in the ice? (Sublimation will also suck heat out of the system, eventually the whole thing should freeze solid then sublimate.)

BTW, a very similar phenomenon is predicted for any Titan ponds of pure methane. They would evaporatively cool, freeze (from the bottom up), then the totally frozen methane pond would slowly sublimate.

Posted by: Eyesonmars Oct 1 2012, 07:27 PM

QUOTE (Juramike @ Oct 1 2012, 06:53 PM) *
(Assuming bp. of H2O at 12 mb is at 10 C)

Water should do the same thing under vacuum, but water's melting point is pretty high, so it will evaporatively cool and then freeze. After it is frozen it will sublimate. Some wierdness might occur if the glass of water is deep enough, the water would freeze on top and maybe you'd get liquid water sealed up in the ice? (Sublimation will also suck heat out of the system, eventually the whole thing should freeze solid then sublimate.)

Do you think it is possible that by exposing the glass to simulated Martian sun the evaporative cooling could be offset enough to keep it liquid until it all evaporated ?

Posted by: Juramike Oct 1 2012, 07:39 PM

Hmmm. That's a good question, but off the top of my head I'd guess no, it wouldn't. Local airspeed is probably a bigger factor, with faster winds increasing sublimation (and evaporation).

Posted by: Eyesonmars Oct 1 2012, 08:00 PM

An interesting experiment I've had students do is to use a syringe to boil water at room temperature and then refrigerate the syringe and redo the experiment.

Posted by: mshell Oct 1 2012, 09:04 PM

I searched the web and couldn't find a phase diagram for water that showed the “area of interest” in much more detail than the one that Udo posted up-thread, so I created my own using Excel and the equations found here:

http://www.iapws.org/relguide/MeltSub2011.pdf (Eqns 1 and 6)

and here:

http://www.iapws.org/relguide/IF97-Rev.pdf (Eqns 29b and 30)

I have attached a couple of phase diagrams for water over the different ranges of temperatures and pressures that have been discussed.





I used the “max pressures” as estimated by Eyesonmars for Hellas and Gale at summer solstice. As djellison said, we are indeed “dancing around a tiny tiny wedge.” I have also attached the values in tabular format:



As Udo noted, if the pressure is below the triple-point of water (6.11657 mBar = 6.11657 hPa = 611.657 Pa), then it doesn't matter the temperature, there can be NO pure liquid water on the surface.

Of course, this all is based on pure water. Impurities (e.g., salts) change both the boiling point (usually elevated) and freezing point (usually depressed) at a given pressure. I couldn't find any easily accessible (and understandable to me) information on how the phase diagram changes with molality, particularly at these low pressures. Maybe someone else can find some Mars-specific info. At Earth pressures, very salty water (think Dead Sea) doesn't freeze until -20 C or lower (from the CRC Handbook of Chemistry and Physics for a 4.6 M solution of NaCl). The boiling point is less affected -- it elevates about +2 C for a 4 M solution.

So, as others have said, liquid water on Mars today seems POSSIBLE, but probably short-lived, if it exists at all.

I’m not going to worry too much more about the fate of glasses of pure water on Mars -- I’m going to put on my brand new, VERY cool, red-cyan clip-on, flip-up glasses and go see if I can talk anyone else in the family into imagining what the waterfalls and rushing waters of Gale Crater could have been like a couple of billion years ago.

Mark

Posted by: djellison Oct 1 2012, 09:09 PM

Beautiful work mshell - that you very much ( and GREAT first post!! ) - that shows the tiny wedge so very well.


Posted by: nprev Oct 2 2012, 12:08 AM

Indeed, I can only echo the Chairman of UMSF's words, Mark...a spectacularly informative & relevant post. Glad you're here! smile.gif

Posted by: marsophile Oct 2 2012, 06:23 AM

That is a fine statement of first-order behavior, but it does not necessarily rule out possible second-order effects provided by such things as surface tension or capillary action in smaller-scale contexts.

Posted by: Harder Oct 2 2012, 07:31 AM

As mshell rightly mentions, the phase diagram is for pure water only. The implication is that in order to assess the equilibrium condition you have to use the partial pressure (for water) at Mars, rather than the absolute pressure. This is linear (every molecule has the same volume) so for agument's sake 1% H2O content in the Mars atmosphere means you have to read the phase diagram at 1% of the prevailing pressure to see what can exist in equilibrium. I think this definitely rules out liquid water.

Posted by: abalone Oct 2 2012, 07:46 AM

What would also be interesting is the depth below ground where the temperature is constantly above Zero and what the pressure would be at this depth. Its possible that only 100s of metres below the surface any water could be permanently liquid. Does anyone have any data in this?

Posted by: vikingmars Oct 2 2012, 07:59 AM

QUOTE (mshell @ Oct 1 2012, 11:04 PM) *
I used the “max pressures” as estimated by Eyesonmars for Hellas and Gale at summer solstice. As djellison said, we are indeed “dancing around a tiny tiny wedge.” I have also attached the values in tabular format:

GREAT post Mshell !
For your info, the max pressure measured by VL2 for its entire mission was 10.72 mb on Sol 277...
=> Could you, please, adjust your last (and nice) table ?
==> Besides, what would be the max pressure at Hellas at Winter solstice ?
(I guess your 14 mb figure is valid for the lowest part of Hellas at -8530m altitude)
Warm welcome and thanks again ! smile.gif smile.gif smile.gif

Posted by: Explorer1 Oct 2 2012, 08:10 AM

On a related note, did anything ever come of the 'brine droplets' on Phoenix's landing legs? I never heard anything about them after the mission ended; if they were salty water than that would be relevant to this discussion.

Posted by: serpens Oct 2 2012, 08:20 AM

QUOTE (vikingmars @ Oct 2 2012, 08:59 AM) *
GREAT post Mshell !
For your info, the max pressure measured by VL2 for its entire mission was 10.72 mb on Sol 277...


Weren't these readings attributed to the diurnal heating and consequential expansion of gas in the Tavis pressure transducers used for the Vikings (and Pathfinder I think), which were assessed as jammed with dust during the landing process? Wasn't this why the Phoenix pressure sensors were not activated until the landing dust had settled?

Posted by: vikingmars Oct 2 2012, 09:31 AM

QUOTE (serpens @ Oct 2 2012, 10:20 AM) *
"...which were assessed as jammed with dust during the landing process?


Well... I was not aware of those being "jammed" nor people doing both experiments for VLs and MPF.

The problem of dust pollution was well foreseen by scientists when designing their instruments before their integration within the landers.
The VL pressure sensors could not be "jammed" by dust, because they were protected from engine exhauts during the landing inside an housing located 1 meter above ground. they were released 5 mn after landing at the end of the meteo boom. And the 1st imaging sequence show that the dust took less than a minute to settle down.
The MPF pressure sensors (derived from the VLs) were protected from dust because they were packed inside the folded petals of the lander, itself protected inside the airbags during its landing on Mars...
So I think that both measurements are perfectly valid, like most scientists involved in both missions. Please, see link :
http://www-k12.atmos.washington.edu/k12/resources/mars_data-information/temperature_overview.html
This is why I think that this "10.72 mb" value is real.
Enjoy ! smile.gif

Posted by: marsophile Oct 2 2012, 03:10 PM

QUOTE (Harder @ Oct 1 2012, 11:31 PM) *
... partial pressure (for water) at Mars, rather than the absolute pressure ...


The partial pressure determines whether evaporation occurs, i.e., vaporization of molecules from the surface of a liquid. The total pressure governs boiling, i.e., vaporization of molecules from the interior of a liquid.

Even on Earth, the partial pressure of water is below the triple point most of the time. Water is generally unstable on Earth's surface, which is why hanging wet clothes out to dry is a reasonable thing to do.

The issue is not whether water is stable (it is not), but whether it is replenished, and how long it can persist. The latter is affected, for example, by the area of the exposed surface, which is why water in a bottle with a narrow top evaporates more slowly than wet clothes.

Posted by: serpens Oct 2 2012, 04:31 PM

QUOTE (vikingmars @ Oct 2 2012, 10:31 AM) *
This is why I think that this "10.72 mb" value is real.
Enjoy ! smile.gif


Thanks. Can't argue with that. On reflection I think my misconception came from a Mars Society presentation a few years ago. I should be more careful in filtering information sources.

Posted by: Tom Dahl Oct 2 2012, 04:50 PM

QUOTE (vikingmars @ Oct 2 2012, 05:31 AM) *
The VL pressure sensors could not be "jammed" by dust, because they were protected from engine exhauts during the landing inside an housing located 1 meter above ground. they were released 5 mn after landing at the end of the meteo boom. And the 1st imaging sequence show that the dust took less than a minute to settle down.

Please forgive me, but I don't think the Viking lander pressure sensor was part of the https://picasaweb.google.com/lh/photo/IU2mMwGbnUfAic6niMYu0NMTjNZETYmyPJy0liipFm0?feat=directlink on the met boom. The MSA had detectors for temperature, wind speed, and wind direction only.

The lander https://picasaweb.google.com/lh/photo/anGhcWpPIZakH65k96t_H9MTjNZETYmyPJy0liipFm0?feat=directlink was located inside the lander body mounted to an interior bracket near leg 2. That https://picasaweb.google.com/lh/photo/vKSkAnUoUTFqkRDohnHkLtMTjNZETYmyPJy0liipFm0?feat=directlink was fed via a tube passing through the lander body to a Kiel Probe located a bit below the lower edge of the body sidebeam. The general arrangement of the sensor and probe are indicated in the following diagram at center-right:

Here is a https://picasaweb.google.com/lh/photo/GyyY8SNRz1XI982vDP2BVNMTjNZETYmyPJy0liipFm0?feat=directlink on the Proof Test Capsule in the Smithsonian NASM. (A few other photos of the probe can be seen via the next and prev PicasaWeb image widgets.) The Flight Capsule 3 (backup) body in the Seattle Museum of Flight has https://picasaweb.google.com/lh/photo/Phg7IbcnsUwyvrsz7cDbENMTjNZETYmyPJy0liipFm0?feat=directlink but not the Kiel Probe itself.

The location of the Keil Probe opening below the lander (and the angle of the probe's cylindrical shroud) was deliberately chosen to enable measurement during the rocket-borne final phase of descent (after aeroshell jettison), as well as while on the surface. Whether the probe opening was susceptible to dust during touchdown may be debatable but it does seem like a possibility. Edited to add: nevertheless, I have no reason to doubt the results obtained.
-- Tom

Posted by: Eyesonmars Oct 2 2012, 06:09 PM

QUOTE (vikingmars @ Oct 2 2012, 08:59 AM) *
GREAT post Mshell !
For your info, the max pressure measured by VL2 for its entire mission was 10.72 mb on Sol 277...

==> Besides, what would be the max pressure at Hellas at Winter solstice ?

As mshell stated, he took the value of 10.2mb from one my earlier posts. I believe i stated that this is representative of the maximum DAILY MEAN surface pressure at the Viking 2 site As the subsequent discussion has shown there is still some uncertainty in what the EXTREME max pressure might have been.
As Vikingmars points out this occurred around LS 280 at the VK2 site. ( I think it was LS and not SOL, as you stated. Correct me if I'm wrong. The max pressure occurred around SOL277 at Vk1 and LS277 at VK2 ) blink.gif

My purpose was to show how difficult it is for liquid H2O to exist ANYWHERE on Mars, let alone at the MSL site.
So using the only long term meteorological surface data we have I estimated what the pressures might have been at Gale and Hellas on that day. ( I even rounded off the scale ht to 11km)

I know this is getting off topic admins so if you think a new topic is warranted I understand..

But back to the Viking data. It is interesting that the max pressure (and wind speed) at the VK2 site occured as a global dust storm engulfed the lander. While this is very pronounced at the VK2 site it is a hardly noticible at the VK1 site.
In addition, the diurnal temperature fluctuations drop to almost nothing by Martian standards ( 10-15c).
Given that the surface can disappear as seen from orbit during a major storm I will go out on a limb and predict that Curiosity will no longer be able to see Mt. Sharp if we have a major/global storm - and for many months afterwords..
If so, our image experts Might have to make due for a while ( not to mention the loss of HiRise imaging ...)

Posted by: Blue Sky Oct 3 2012, 01:53 AM

It is interesting that in the NASA presentation of the fan outflow pattern last week, there was no discussion of where all that water came from in the first place. Based on the location of Gale crater, it would seem to have been from the northern ocean that was assumed to exist in the Noachian age. A long time ago.

Posted by: abalone Oct 3 2012, 03:17 AM

An interesting paper
http://online.liebertpub.com/doi/pdfplus/10.1089/ast.2011.0660
Liquid water could be stable as little as 100-200m below the surface
and
"At temperatures below 0C, liquid water can exist as either
a thin film or a brine with a freezing point < 0C. Including
salts to estimate ‘‘average Mars salinity water’’ expands the
regions where liquid water can occur. Modeling by Mellon
and Phillips (2001) showed that a concentration of 15–40% of
salts (sulfates, chlorides, bromides, carbonates, and nitrates)
is sufficient to allow the melting of ground ice (or to maintain
liquid water) in the top few meters of soil."

Posted by: vikingmars Oct 3 2012, 08:15 AM

QUOTE (Eyesonmars @ Oct 2 2012, 08:09 PM) *
As Vikingmars points out this occurred around LS 280 at the VK2 site. ( I think it was LS and not SOL, as you stated. Correct me if I'm wrong. The max pressure occurred around SOL277 at Vk1 and LS277 at VK2 ) blink.gif

...The max pressure occurred really on Sol 277.34 but the Ls is 279.93... You are right, not far from this value wink.gif (see table herebelow as "pdf" file)
 VL2_Sol277.pdf ( 27.43K ) : 715

Posted by: mshell Oct 5 2012, 07:44 PM

This is an update of my up-thread post http://www.unmannedspaceflight.com/index.php?s=&showtopic=7489&view=findpost&p=192563 where I created phase diagrams for pure water and speculated (with input from Eyesonmars) on the maximum atmospheric pressures that we might see at Gale and on Mars as a whole. I spent a couple of blissful hours last night running down elevation figures, Viking Lander weather data, REMS data, and concepts such as “scale height”.

Here’s what I found …

1. Eyesonmars stated in his post http://www.unmannedspaceflight.com/index.php?s=&showtopic=7489&view=findpost&p=192448 that the MSL site was “another 2km or so LOWER” than the Viking 2 landing site at Utopia Planitia. We can now update this based on the latest Mars Orbiter Laser Altimeter (MOLA) data. It was interesting to see that the original Viking elevation data was based on a different “reference ellipsoid” from what is now used for MOLA elevations. The MOLA elevations are based on a Mars geoid (an “areoid”) with a radius equal to the average equatorial radius of Mars and with a surface that has an equipotential gravitational field. This is just what we need to compare barometric pressures. It turns out that Curiosity’s MOLA elevation is virtually the SAME as the Viking 2 site:



The MOLA elevations are accurate to about +/- 1m. The references can be found on pp. 11-12, http://www-geodyn.mit.edu/mola.summary.pdf.

I took the Curiosity elevations from the excellent profile map attached to http://www.unmannedspaceflight.com/index.php?s=&showtopic=7413&view=findpost&p=189181 by pgrindrod, which I am virtually positive are based on MOLA elevation data, since it seems to agree with other sources.

2. Since the elevations are virtually identical, it seems reasonable to expect that the barometric pressures at Gale will be similar to those at Viking 2. I found the weather data for Viking 2 http://www-k12.atmos.washington.edu/k12/resources/mars_data-information/data.html and the REMS weather data for MSL http://marsweather.com/data for daily averages. You can press the “Data” button at the bottom of the weather display to access the (approximately) hourly data from Sols 9-12.

The Viking data was easy to cut and paste into a spreadsheet, but I had to type the REMS data in by hand. Does anyone know a place to get ALL of the REMS readings? Also, please contact me if you are interested in getting the weather data spreadsheet. The max average daily value for Viking 2 was indeed 8.20 mBar as Eyesonmars reported, and the single maximum pressure reading is indeed 10.72 mB as Vikingmars reported http://www.unmannedspaceflight.com/index.php?s=&showtopic=7489&view=findpost&p=192592.

I then plotted the average daily pressures from Viking against the Solar Longitude (a measure of the “season” on Mars) and overlaid Curiosity’s REMS average daily pressures:



The data are right on top of each other, which gives some credence to the concept that the atmospheric pressures at Gale through the summer are likely to be similar to what we have already seen with Viking 2 at Utopia Planitia. This leads me to estimate that we could very well see pressures above 10 mB at Gale.

3. With the updated elevations and using a “scale height” of 11.1 km (which varies a bit with temperature and is referenced http://nssdc.gsfc.nasa.gov/planetary/factsheet/marsfact.html), we get a maximum pressure at the lowest point on Mars (an impact crater in Hellas Basin, see page 11 of http://www-geodyn.mit.edu/mola.summary.pdf) of almost 15mB -- which is a bit more than the 14 mB that Eyesonmars had previously estimated. Although, given that Hellas is not in the “tropics” of Mars, the temperatures are likely to stay well below freezing year-round. Viking 2, for example, never got above -20 C.

Here is the updated phase diagram:



4. The most interesting thing that I found during my investigation was http://www.europlanet-eu.org/outreach/index.php?option=com_content&task=view&id=392&Itemid=41 press release from the principal investigator of the REMS instrument, who says that “in the daytime, we could see temperatures high enough for liquid water on a regular basis” (my emphasis).

Maybe, just maybe, djellison’s “tiny tiny wedge” at the bottom of the phase diagram is just big enough …

Mark

Posted by: Explorer1 Oct 5 2012, 09:10 PM

Very impressive work Mark!
So the pressure will tick up and up until the start of summer, but what happens then? Would a gas sample in SAM see any difference?

Posted by: mshell Oct 11 2012, 04:43 AM

In a private message to me, Eyesonmars indicated that:

QUOTE
... southern summer daytime temperatures (are) about 30c warmer than the equivalent latitude in the northern hemisphere.

In my previous post, I had made the erroneous assumption that Mars would be symmetrical with regards to average temps at a given latitude. I knew that there is a big average elevation asymmetry between north and south, but I didn't know about the temperature difference. I have since learned that the difference is caused primarily by the high orbital eccentricity, whereby Mars is significantly closer to the sun during southern summer compared to northern summer.

With a little digging I found the TES (Thermal Emissions Spectrometer) data from the MGS (Mars Global Surveyor) mission. http://planetologia.elte.hu/mcdd/index.phtml?cim=held.html shows clearly that the daily max temperatures are well above freezing in the Hellas Basin during many days in the local summer, just as Eyesonmars pointed out.

I apologize for the mistake -- I should have been more careful.

It seems clear to me that the pressure/temperature conditions are such that, at least in some places and at some times of the year (including Gale and Hellas), liquid water COULD exist during the day on the surface of Mars -- although, as has already been mentioned up-thread, the evaporation rate would likely be very high. And of course, this is only relevant IF there is any water left to be liquid.

Mark

Posted by: Explorer1 Oct 11 2012, 06:38 AM

There's also the fact that the Martian axis is more tilted than Earth's, at 25 degrees, so the seasons are more extreme than our own. But without a big moon to stabilize, it's obviously subject to change.

Posted by: Eyesonmars Oct 11 2012, 06:05 PM

Calculations suggest that the obliquity of Mars varies greatly with a period of about 100,000 years.
During minimum obliquity, with Mars standing more or less straight up on its axis, it has been shown that much if not all of the CO2 in the south polar cap returns to the atmosphere. Recently MRO discovered vastly more CO2 in the south polar cap than what was previously thought. You can read about it http://www.universetoday.com/85132/mars-underground-atmosphere/#ixzz290rrqA1m

QUOTE
While scientists were aware of seasonal CO2 ice layers atop the water ice this new discovery brings to light nearly 30 times more frozen CO2 than was previously believed to exist. In fact this particular deposit alone contains 80% the amount of CO2 currently present in the planet’s entire atmosphere.


This means, looking back at mshell's phase diagram, pressures as high as 27mb at Hellas and 18mb at Gale are then likely on this time scale. This puts the boiling point of water as high as 22c at Hellas - room temperature.

A less hostile environment for liquid H20 means less liquid water is initially required to form water related geologic features.

Delaying or slowing down the phase change from the liquid state will give any outflow more time to do its work

Posted by: Gsnorgathon Oct 11 2012, 07:14 PM

QUOTE (Eyesonmars @ Oct 11 2012, 10:05 AM) *
... During minimum obliquity, with Mars standing more or less straight up on its axis, it has been shown that much if not all of the CO2 in the south polar cap returns to the atmosphere. ...

Do you have a link for that? My understanding is that minimum obliquity would result in minimum insolation at the poles, making them cold traps and thus reducing atmospheric pressure significantly.

Posted by: mcaplinger Oct 11 2012, 08:23 PM

QUOTE (Gsnorgathon @ Oct 11 2012, 12:14 PM) *
My understanding is that minimum obliquity would result in minimum insolation at the poles, making them cold traps and thus reducing atmospheric pressure significantly.

Yes. See, e.g., http://spacescience.arc.nasa.gov/mars-climate-workshop-2012/documents/extendedabstracts/Wood_SE_ExAbst.pdf

QUOTE
At low obliquity the polar regions of Mars receive less annual insolation and can reach a
point where the total CO2 sublimation at the pole becomes less than the total condensation – forming a
perennial CO2 ice polar cap. Below this critical obliquity the mass of the CO2 polar cap(s) increases at the
expense of the atmosphere, potentially leading to atmospheric “collapse”.


Posted by: Eyesonmars Oct 11 2012, 08:45 PM

Of course, I do have it backward. Atmospheric mass is maximized at larger obliquidity
Thank you for the correction
http://www.google.com/imgres?q=martian+obliquity&start=129&hl=en&sa=X&rlz=1C1CHFX_enUS382US382&biw=1024&bih=475&tbm=isch&prmd=imvns&tbnid=hqRtjLZvSUwVCM:&imgrefurl=http://cmex.ihmc.us/data/frontpg/missions/samret/haberle/rmh_abs.htm&docid=fzprD-yKnwJmvM&imgurl=http://cmex.ihmc.us/data/frontpg/missions/samret/haberle/RMH_VG16.GIF&w=638&h=397&ei=Jy13UO7TOuKFywGF-ICoDg&zoom=1&iact=hc&vpx=83&vpy=187&dur=22947&hovh=177&hovw=285&tx=146&ty=94&sig=111159900871532310818&page=10&tbnh=106&tbnw=170&ndsp=15&ved=1t:429,r:10,s:129,i:136 for example

But does it change the result ?
Any martian climate modelers out there ?

Posted by: Eyesonmars Oct 11 2012, 09:14 PM

QUOTE
Although the albedo effect resulted in thick
perennial CO2 caps during some high obliquity periods, there were several times when the atmospheric
pressure reached 1000-1200 Pa followed by a rapid
drop to <100 Pa


This is a quote from the above paper ( thanks to mcaplinger)

also - from Fig. 8 it appears, at least from this model, that these brief periods of higher atmospheric density do not happen every 100,000 years but have occured several times in the last 1,000,000 years

Posted by: serpens Oct 11 2012, 10:06 PM

I've never been able to get my mind around the very high obliquity effects. Wouldn't this result in an increased transfer of CO2 between poles on an annual basis. Large pressure swings between hemispheres but with limitations on the CO2 actually retained in the atmosphere in gaseous state?

Posted by: climber Oct 14 2012, 06:26 PM

I watched the jump record of Baumgartner a few minutes ago! Pressure at 39 kms or so was 5mb...

Posted by: MichaelT Nov 1 2012, 09:46 AM

Dear all,

I am trying to find out if it is possible to access the REMS data files somewhere. The data are displayed http://cab.inta-csic.es/rems/marsweather.html and http://marsweather.com/data and you can also view past data on the latter page. However, instead of copying all these values by hand, it would be great to be able to read the REMS data from the XML files directly and automatically to create a time series of e.g. the max and min temperatures.
I found that the current data are located in this XML file: http://cab.inta-csic.es/rems/rems_weather.xml

Does anybody know if it possible to access past data as well? I could not find any links on the respective websites.

Cheers,
Michael

Posted by: marsophile Nov 1 2012, 03:37 PM

QUOTE (MichaelT @ Nov 1 2012, 02:46 AM) *
Does anybody know if it possible to access past data as well?


If you view the page source for marsweather.com in html and follow some links, you can find the Sol data in xml format at http://data.marsweather.com/rems_climate.xml, but you may want to check with marsweather.com as to what their policy is on credit notices if you post a rendering of the data elsewhere. You can post a message on their blog asking questions like that.

Posted by: MichaelT Nov 1 2012, 04:16 PM

Thanks Marsophile! cool.gif

That is exactly what I had been unable to find! I do not plan to post diagrams. It is purely out of personal interest. Maybe I will will use a graph for a presentation in a few weeks where I will certainly acknowledge the source of the data. Otherwise I will ask just as you recommend.

Cheers
Michael

Posted by: PaulH51 Mar 6 2013, 09:23 AM

Has anyone been able to extract usable REMS data from the first MSL PDS issue (Sols 0-89)?

Edit : I can locate TAB files (eg ACQ_M1) and can open these in EXCEL, but they appear to only contain START_BYTE and BYTE data according to the associated FMT file, am looking for air and ground temperatures / pressure / wind speed etc. I am probably barking up the wrong tree, and I may have to convert the BYTES into values, but have been barking up that tree for most of the day without a lot success (so far anyway)

Apologies if this is in the wrong thread...

EDIT : Thanks for feedback djellison and jmknapp

Posted by: djellison Mar 6 2013, 04:25 PM

No - as the PDS release clearly states, they are releasing raw data - not the calibrated science data.

It is - as yet - of no use to us.

Posted by: jmknapp Mar 6 2013, 08:05 PM

The columns are listed in this file:

http://atmos.nmsu.edu/PDS/data/mslrem_0001/LABEL/ACQ.FMT

Looks like signed integers mostly with negative values given as 32768-65535.

The associated data file is the biggest one in each folder, e.g., for sol 89:

http://atmos.nmsu.edu/PDS/data/mslrem_0001/DATA/SOL_00001_00089/SOL00089/RME_405347393ESE00890000000ACQ____M1.TAB

Posted by: fredk Mar 6 2013, 08:57 PM

Mods - shouldn't these last few posts go in the PDS thread?



ADMIN NOTE: Mmm? Probably OK here as the original question and subsequent answers are pertinent to the topic.

Posted by: jmknapp Mar 7 2013, 02:14 AM

Took a look at an air temperature curve from the uncalibrated REMS data.

Sol 10 data file (comma-separated): http://atmos.nmsu.edu/PDS/data/mslrem_0001/DATA/SOL_00001_00089/SOL00010/RME_398334213ESE00100000000ACQ____M1.TAB
Column descriptions: http://atmos.nmsu.edu/PDS/data/mslrem_0001/LABEL/ACQ.FMT

Extracting the columns for the two boom air temperature sensors B1_AIR_Temp and B2_AIR_Temp yields this plot for sol 10:



I picked sol 10 because it can be compared with a calibrated air temperature plot released during one of the telecons:

http://www.nasa.gov/mission_pages/msl/multimedia/pia16081.html

Not sure what's going on with Boom 2, but comparing the Boom 1 curve with the calibrated curve shows a good match shape-wise up to sol 10.6. At that point it looks like for some reason they drew a straight line to the start of sol 11.

Anyway, since the minimum of the calibrated curve is about -90°C and the maximum about 3°C, and the min/max of the uncalibrated curve is -9512/2028, a rough formula to convert the Boom 1 uncalibrated values to °C is:

T = 0.00806*n - 13.3

Posted by: PaulH51 Mar 7 2013, 07:20 AM

QUOTE (jmknapp @ Mar 7 2013, 10:14 AM) *
Took a look at an air temperature curve from the uncalibrated REMS data...

Many thanks for pointing me in the right direction Joe. Hopefully the anomalies on the temperature values from the other boom are not an indication of more issues with the REMS instrument package. Hopefully we won't have to wait too long for the calibrated data to be issued.

Posted by: djellison Mar 7 2013, 12:23 PM

QUOTE (PaulH51 @ Mar 6 2013, 11:20 PM) *
Hopefully we won't have to wait too long for the calibrated data to be issued.


Again -as the PDS release schedule states
"Part 2, March 20, 2013, will include the derived data products (RDRs) for Sols 0 though 89 for the APXS, ChemCam, DAN, Hazcam, Navcam, and REMS instruments, along with both the EDRs and RDRs for the CheMin and RAD instruments, and the RDRs for the SAM instrument."


Posted by: jmknapp Mar 7 2013, 12:56 PM

March 20th would put it during their upcoming conference.

In the meantime, here's a page for showing Boom 1 temperature plots for sols 1-89, using the rough conversion above to convert to Celsius:

http://curiosityrover.com/b1air.php?sol=1

Posted by: PaulH51 Mar 7 2013, 01:37 PM

Thanks Joe, I envy your skills in extracting the data and creating those multi-page charts,

I compared around 10 of the sols with the min / max levels reported by the web based Mars weather reports. No precise matches (especially on the 'min' values), not all together surprising as I have long suspected the standard of what they have reported. Thankfully the 20th is very close, I look forward to seeing the calibrated data from NASA / JPL..

Thanks again Joe

Posted by: djellison Mar 7 2013, 03:28 PM

The REMS instrument isn't a NASA/JPL instrument - it's from the Centro de Astrobiología, Spain

Posted by: jmknapp Mar 7 2013, 09:01 PM

It'll be interesting to compare to the calibrated results to see how involved that process is, or if it's just linear. One thing that strikes me is the frequent spikes/dips of up to about 10 degrees over a short period of time. Could these be "real"? Maybe warmer air from the RTG fins wafts over from time to time?

Posted by: PaulH51 Mar 7 2013, 11:44 PM

QUOTE (djellison @ Mar 7 2013, 11:28 PM) *
The REMS instrument isn't a NASA/JPL instrument - it's from the Centro de Astrobiología, Spain

Spanish instruments, but the REMS data (according to the Centro de Astrobiología web site) will be issued by NASA /JPL through its PDS portal.

Posted by: CosmicRocker Mar 8 2013, 08:33 AM

QUOTE (jmknapp @ Mar 7 2013, 06:56 AM) *
... here's a page for showing Boom 1 temperature plots for sols 1-89, using the rough conversion above to convert to Celsius:

http://curiosityrover.com/b1air.php?sol=1

That is so cool. Might you be willing to create a way plot the data across multiple sols?

Posted by: Gerald Mar 8 2013, 12:57 PM

QUOTE (jmknapp @ Mar 7 2013, 10:01 PM) *
... One thing that strikes me is the frequent spikes/dips of up to about 10 degrees over a short period of time. Could these be "real"? Maybe warmer air from the RTG fins wafts over from time to time?

A hint may give, whether shorter-term changes of air temperature is correlated with temperature in the interior of the rover.
I assumed, that the ASIC values measure some interior temperature of the rover.
Therefore I put the three sol 11 uncalibrated temperature curves (ASIC 1, ASIC 2, and Boom 1 Air) into one diagram (the first by timestamps, the second by records), the more jiggered are the ASICs.

To my eyes there is some correlation of temperature changes between ASICs and Air, best visible in "the evening" of the second diagram, although I didn't investigate this quantitatively.
This may support your idea to a certain degree. But it doesn't exclude influence of "true" short-term changes of air temperature.

EDIT: I'm a bit confused, because http://atmos.nmsu.edu/PDS/data/mslrem_0001/DATA/SOL_00001_00089/SOL00010/RME_398334213ESE00100000000ACQ____M1.LBL is internally called sol 11 (MSL:LOCAL_MEAN_SOLAR_TIME = "Sol-00011M10:08:40:097"), but it's in subdirectory for sol 10. So there seem to be two different counters for sols.

Posted by: SpaceListener Mar 8 2013, 03:25 PM

QUOTE (jmknapp @ Mar 7 2013, 04:01 PM) *
It'll be interesting to compare to the calibrated results to see how involved that process is, or if it's just linear. One thing that strikes me is the frequent spikes/dips of up to about 10 degrees over a short period of time. Could these be "real"? Maybe warmer air from the RTG fins wafts over from time to time?

Yes it is a good question since the mini-variations of temperature only occurs anything different than the sun's heat.

Posted by: jmknapp Mar 8 2013, 06:27 PM

Gerald, it may be correlated because the ASIC chip isn't very far into the interior of the rover, according to this diagram:



That's from this document:

https://www.google.com/url?sa=t&rct=j&q=&esrc=s&source=web&cd=1&ved=0CDIQFjAA&url=http%3A%2F%2Fmicroelectronics.esa.int%2Famicsa%2F2008%2Fspaceapplications%2FCHICCA%252002%2520SEPTEMBER%25200915.ppt&ei=Vik6Uaf4GIWz0QGgzoHwBQ&usg=AFQjCNEKXPq8wthdEk6z4hf3faFeVMlepw&sig2=WbHoROPyDhk86e6dxzgl7g&bvm=bv.43287494,d.dmQ

Showing the trend over multiple sols would take some data massaging, as each day can have something like 30,000 samples, which would be a lot of overkill. There are some apparent glitches here and there that might have to be dealt with. Maybe best to wait for the calibrated data!

Specifically the ground temperature daily swing would be interesting--isn't that what they used to show the transition between the high and low thermal inertial units as they entered Glenelg?

Posted by: Gerald Mar 8 2013, 07:30 PM

You are right: It's hard to tell, in which way the counter values are composed. Looks, as if we had to wait for calibrated data to assess those small fluctuations.
Ground temperatures will also be the more interesting data to estimate the possibility of liquid water at Gale. Probably the thermopile data will be better-suited for this purpose.

And thanks for pointing to the docu of the locations of the sensors!

Posted by: jmknapp Mar 12 2013, 10:21 AM

Here's a quick-and-dirty chart of the temperature over sols 1-89:



Actual calibration may vary, but we can safely say that, baby, it's cold up there.

Another potentially interesting way to look at the data is to average the sols together to get an average temperature curve over the course of the day:



That's a little dodgy because the sun rise and set times vary over the time period, but not by much:

sol 0: rise 5:31 set 17:23
sol 89: rise 5:18 set 17:24

Posted by: jmknapp Aug 14 2013, 10:46 AM

A couple of plots generated from the http://data.marsweather.com/rems_climate.json, which appears to be kept current within a few sols:

http://curiositymsl.com/rems/temps.html
http://curiositymsl.com/rems/pressure.html

Those are set up to automatically update from the feed every day.

Posted by: vikingmars Aug 15 2013, 09:46 AM

QUOTE (jmknapp @ Aug 14 2013, 12:46 PM) *
Those are set up to automatically update from the feed every day.

Thanks a lot for those useful links and for this real good work smile.gif
For the temperatures, could they be generated, please, also with °Celsius (or °Kelvins) : this could really be of a great help for the rest of the planet using the metric system...
Thanks again so much and cheers ! wheel.gif wheel.gif wheel.gif

Posted by: PaulH51 Aug 15 2013, 10:02 AM

QUOTE (jmknapp @ Aug 14 2013, 06:46 PM) *
http://curiositymsl.com/rems/temps.html http://curiositymsl.com/rems/pressure.html
Those are set up to automatically update from the feed every day.

Joe,
great work as always, will you be adding links to these charts and data on your rover links page? (http://curiosityrover.com/links.html)
Paul.

Posted by: jmknapp Aug 15 2013, 12:16 PM

Paul-- Good idea--I was trying to see how I could fit another button ("REMS weather") on the main page, but for now I put the urls in the links page.

VikingMars-- Aww, I suppose you also want the pressure graph in Pascals. smile.gif I'll work on adding a selectable dropdown for C, F, K.

As for the data itself, it's been mentioned before, but still very interesting that if you take the phase chart of water (with the triple point, etc.) it looks like there are times at this site when liquid water could theoretically exist on the surface.

Posted by: jmknapp Aug 15 2013, 03:57 PM

OK, here are the plots with selectable units:

http://curiosityrover.com/rems/temps.php
http://curiosityrover.com/rems/pressure.php

Posted by: centsworth_II Aug 15 2013, 09:23 PM

QUOTE (jmknapp @ Aug 15 2013, 10:57 AM) *
OK, here are the plots with selectable units...
One thing that leaps out on the Centigrade scale is how the freezing point of water looks like a rough mean for the highs. The lack of a 32 degree line makes it less obvious on the Fahrenheit scale. Never heard of the Rankine scale before. Interesting.

Posted by: jmknapp Aug 16 2013, 07:02 PM

I put in a dotted line to mark the H2O freezing point for all the scales. Also a button on the main page to see the weather (temperature and pressure anyway) on one page: http://curiosityrover.com/rems

Posted by: Explorer1 Aug 16 2013, 08:16 PM

Looks like the colors at around sol 73 or so are switched, with the max in blue and minimum in red; just a glitch?
Very nice chart otherwise, it will become more and more interesting as long term observations come in.

Posted by: jmknapp Aug 16 2013, 08:30 PM

QUOTE (Explorer1 @ Aug 16 2013, 04:16 PM) *
Looks like the colors at around sol 73 or so are switched, with the max in blue and minimum in red; just a glitch?
Very nice chart otherwise, it will become more and more interesting as long term observations come in.


That's the way it is in the data file from http://data.marsweather.com/rems_climate.xml (sol 73). I was thinking of filtering that out, but there's also the weird dip lasting several sols around sol 295, which can't be real (can it?). So maybe it's best to just leave it as is.

The pressure plot is interesting because of the local minimum of pressure--like there will be at least two minimums a year, assuming it's due to the solar cycle and not some other factor.

Posted by: Harder Aug 16 2013, 09:17 PM

The dotted line at 32 F is too pessimistic when you're thinking about liquid water on Mars. A reasonable quantity of dissolved (soluble) salts would easily drop the freezing point to 25 F or lower, which brings most of the measured temps "above the line".

But pls. bear in mind that all the mentioned triple point- and other liq-vap data for water are measured in what's called a "closed system", i.e. 100% water inside a cylinder with a moving piston. An equivalent closed system at Mars would have to be underground. At the surface of Mars we have an open system with such an extremely low partial pressure of water (%water vapor in Mars atmosphere * abs. pressure at the surface of Mars) that we're nowhere near liq/vap equilibrium. Any water pushed to the surface of Mars would "explode" into vapor.


Posted by: Deimos Aug 16 2013, 10:13 PM

QUOTE (jmknapp @ Aug 16 2013, 08:30 PM) *
The pressure plot is interesting because of the local minimum of pressure--like there will be at least two minimums a year, assuming it's due to the solar cycle and not some other factor.

Right now, northern polar CO2 is starting to sublimate. Within a couple months, it'll be all gone and condensation in the south will dominate until southern spring approaches and the Sun begins to rise on the Southern seasonal cap. Maybe a local maximum around 8.8-8.9, around 460 or so?

Posted by: jmknapp Aug 16 2013, 11:54 PM

QUOTE (Harder @ Aug 16 2013, 05:17 PM) *
Any water pushed to the surface of Mars would "explode" into vapor.


Guess I don't follow--isn't the (low) pressure already accounted for in the phase diagram?

Seems like a lot of people have mentioned that the conditions on Mars are just on the cusp of the wet side of the triple point, like http://science1.nasa.gov/science-news/science-at-nasa/2000/ast29jun_1m/:

QUOTE
The air pressure is so low on Mars that even in the most favorable spots, where the pressure is higher than average, liquid water is restricted to the range 0 to +10 °C," says Bob Haberle of the NASA/Ames Research Center. ...

"First of all, you have to remember that the average atmospheric pressure on Mars is very close to the triple point of water," explains Richard Hoover, an astrobiologist at the Marshall Space Flight Center. "You only have to increase the pressure a little bit to make liquid water possible."

The 'triple point' is the combination of pressure (6.1 millibars) and temperature (0.01 °C) at which water can exist simultaneously in all three states: a solid, a liquid and a gas (see the 'phase diagram' below). ... On Mars the globally-averaged surface pressure of the planet's atmosphere is only slightly less than 6.1 millibars.

"That's the average," says Haberle, "so some places will have pressures that are higher than 6.1 millibars and others will be lower. If we look at sites on Mars where the pressure is a bit higher, that's where water can theoretically exist as a liquid."


It may evaporate, but necessarily "explode"? For example, the high temperatures for sols 100-120 were between 0 and 10 °C and the pressure about 8.5 mbar.

Posted by: Harder Aug 17 2013, 04:11 PM

The term "explode" came to my mind from 1st hand experience working in an oil refinery: in a so-called vacuum tower, a huge distillation tower with trays (internals) designed to separate heavier oil fractions under medium vacuum conditions, it is customary to use steam as a "stripping medium" i.e. supply steam to the bottom of the tower to make sure that lighter components are maximally recovered from the asphalt-like bottom product of the vacuum tower.

A well-known pitfall during start-up of such a column is the situation where the stripping steam is not dry and superheated but "wet". When liquid water enters the vacuum tower through the stripping steam inlet it literally "explodes" due to its huge volumetric expansion from liquid to vapor, damaging the internals of the vacuum tower in the process. The analogy is in fact not to bad, because distillation towers also have other components (molecules) in the vapor phase, similar to N2, CO2 etc in the vapor phase (atmosphere) of Mars.

But on Mars the situation is much more extreme: the vacuum is much better that in my oil refinery! So I'm really confident that any liquid water being pushed to the surface of Mars will instantly vaporize. The only possible exception being water in the from of brines where the freezing point as well as absolute quantity of water in the brine may be so low that indeed "explode" is an over the top characterization. Sorry for that. Another factor that indeed makes "explode" less likely is that the very substantial heat of vaporization that is necessary to vaporize water is readily supplied in the distillation column (with devastating effects!) but not so on Mars. So when water vaporizes on the surface of Mars it has to withdraw the heat of vaporization from its environment which may slow (but not halt) the vaporization process.

I hope there are other chemical engrs in this forum who may chip in and corroborate, but the scientific bottom line is unfortunately firmly against any liquid water on the surface of Mars due to the extremely low pressure. The equilibrium equations for H2O simply dictate vapor phase, I'm sorry if I pour "cold water" (pun intended) on the hopes and aspirations of other forum members!

Regards,
Peter

Posted by: djellison Aug 17 2013, 04:45 PM

QUOTE (jmknapp @ Aug 16 2013, 04:54 PM) *
the high temperatures for sols 100-120 were between 0 and 10 °C and the pressure about 8.5 mbar.


At those pressures, whilst we are above the triple point we are in the realm of a very narrow wedge for the fluid phase. The difference between melting point and boiling point at those pressures is less than 10 degrees. Indeed - those 'hotter' days are almost certainly well above the boiling point.

Hence why the occasional fluid flows seen in crater walls have been described as a boiling torrent. That's what they would look like.

For a crude playground of values...
http://www.trimen.pl/witek/calculators/wrzenie.html

As an example, you can put in the 760mm pressure and 100 degC for water then put in your own pressure.

8mm of pressure ( >10mb ) has a boiling point of only 4 degrees.

So - do we reach the melting point of ice. Yup. And almost every time we do, we probably go over the boiling point as well.

Posted by: Explorer1 Aug 17 2013, 04:46 PM

The lowest regions of the planet do have higher pressures; Hellas basin's bottom is at over 11 mb, and we know Martian summer days can get above freezing as Doug said (milder given the southern locale). Obviously Hirise hasn't noticed anything, and there has been no landing with something like REMS for ground truth. If there's ice hidden under the surface, like Phoenix saw, a trench at the right time might have results...

Posted by: djellison Aug 17 2013, 04:55 PM

QUOTE (Explorer1 @ Aug 17 2013, 08:46 AM) *
like Phoenix saw


Indeed - and the next time we looked, it was gone.

Posted by: jmknapp Aug 17 2013, 05:55 PM

QUOTE (Harder @ Aug 17 2013, 12:11 PM) *
I hope there are other chemical engrs in this forum who may chip in and corroborate, but the scientific bottom line is unfortunately firmly against any liquid water on the surface of Mars due to the extremely low pressure. The equilibrium equations for H2O simply dictate vapor phase, I'm sorry if I pour "cold water" (pun intended) on the hopes and aspirations of other forum members!


More to the point, perhaps you are throwing cold water on the afore-quoted NASA scientists!

But how about an actual experiment--namely from this paper:

http://www.lpi.usra.edu/publications/reports/CB-1063/UCB.pdf

The authors note a possible ambiguity in interpreting the phase diagram:

QUOTE
A question frequently asked is whether the abscissa in Figure 3 is total pressure or partial pressure of water vapor. If the former, the pressure on Mars is frequently above the triple point. If the latter, the pressure would always be below it since the partial pressure of water vapor in the atmosphere is only a fraction of a millibar. This question will be addressed in the experimental methods section of this paper.


They simulated the Mars environment as follows:

QUOTE
Simulating Martian conditions in a bell jar was the objective of the experimental phase of this study. An ice cube in a glass funnel placed inside a bell jar containing Drierite (a desiccant), calibrated thermometers, and dry ice (to create a CO2 atmosphere) was kept under Martian pressures by a vacuum pump.


Result:

QUOTE
At a mean ice temperature of 0°C, as seen in figure 8, liquid water was observed at pressures between 3 mb and 10 mb, Mars like conditions. This data demonstrates that liquid water can exist under these simulated Martian conditions. ...

The purpose of the bell jar experiment was to determine the feasibility of liquid water under Martian conditions. This condition was met. Additionally, we can conclude that total pressure drives the phase change of water, not the partial pressure of water vapor in the atmosphere.


After all, even though there's almost no water in the Mars atmosphere, there are still CO2 molecules banging against the surface of any liquid. Maybe where you go wrong is focusing too much on the equilbrium state. If I put a dish of water out in my back yard, the equilibrium state is clearly the vapor phase, although it will take some time to get there.

Posted by: Explorer1 Aug 17 2013, 06:23 PM

That's the point I was making as well; unlike the 'typical' Mars conditions that Phoenix encountered at Vastita, subsurface ice at Hellas (if it exists) could melt if a trench exposed them one warm summer day, at least until nightfall.
This is all academic until someone actually does the experiment of course; none of Exomars' candidate landing sites include that area...

Posted by: Harder Aug 17 2013, 08:28 PM

I stand corrected.. Where I went too fast was in assuming that ground ice does not exist because it is not stable in the present time and that liquid water could thus only feature in transient events - measured in hours (as I read in Wikipedia) - with thermodynamics inexorably determining the final outcome.

Many thanks for the reference, Joe, I've now also read the article "On the possibility of liquid water on present-day Mars" by Mr. Haberle et al. I recommend everybody interested in the subject to look it up and read it. It also makes a strong case for further experiments - as brought up by Explorer1.

Posted by: Explorer1 Aug 17 2013, 10:16 PM

I do wonder if the Exomars rover's drill would be suitable for such an examination. It's designed to go a lot deeper than any scoop, but it's a corer; so material wouldn't really be exposed per se. In any case, a low elevation is still the main requirement.

Posted by: serpens Aug 17 2013, 11:28 PM

It is a little difficult to correlate the theories and simulation experiments regarding liquid water on Mars with the presence of kieserite. Either there is something about the stability field of kieserite under Martian conditions that we do not understand or the simulations/theories on water on Mars are missing something.

Posted by: jmknapp Aug 17 2013, 11:39 PM

QUOTE (Harder @ Aug 17 2013, 04:28 PM) *
I've now also read the article "On the possibility of liquid water on present-day Mars" by Mr. Haberle et al. I recommend everybody interested in the subject to look it up and read it. It also makes a strong case for further experiments - as brought up by Explorer1.


Yes, that http://funnel.sfsu.edu/students/roestg/Courses/Students/Carrie%20Popins/Fall%202012%20(NOW)/795/haberle%20water%20on%20mars%202001plw.pdf is written very clearly! If I read it right, they don't identify the Gale Crater region specifically as having the amount of "degree days" conducive to liquid water, but the date of it is 2001 and to date quite a bit more is known about the Mars climate and of course there's the ground truth provided by Curiosity.

Posted by: Gerald Aug 18 2013, 12:29 AM

If, as it looks, perchlorates are more or less ubiquitious at the Martian surface, is it realistic to look for pure liquid water on Mars?
As perchlorates are highly hygroscopic, I'd expect water to be bound either as crystal water to perchlorates or to form a brine in the sense of http://www.lpi.usra.edu/meetings/lpsc2009/pdf/2420.pdf. Such brines may stay liquid down to -70°C.
So, I'd suggest it's worth to think about phase diagrams containing (perchlorate) brines and water bound in crystals under equilibrium conditions with water vapor partial pressure in the atmosphere. I'd guess, that the water vapor partial pressure needed for an equilibrium with a perchlorate brine at low temperature is much lower than for pure water at the triple point.
The Phoenix results about perchlorates weren't yet available in 2001. Therefore http://funnel.sfsu.edu/students/roestg/Courses/Students/Carrie%20Popins/Fall%202012%20(NOW)/795/haberle%20water%20on%20mars%202001plw.pdf looks a bit obsolete to me.

Posted by: djellison Aug 18 2013, 01:49 AM

QUOTE (Gerald @ Aug 17 2013, 04:29 PM) *
Such brines may stay liquid down to -70°C.


What's its boiling point?

Taking brine as an example - that can reduce the freezing point (at 1 atmos) to -21c. However, boiling point goes up just a couple of degrees.

So - frequent weather conditions on Mars would exceed the melting point of such brines...rendering liquid water possible on the surface.....and then rapidly exceed the boiling point ( which is likely to be single digit degC at Martian air pressure ) rendering the water transient at best



Posted by: Gerald Aug 18 2013, 03:52 AM

At 10 mbar the boiling point of a saturated Mg(ClO4)2 solution is about 295K (using 10mbar partial pressure at 295K as estimate for the boiling point, according to the light brown cross in figure 3 of http://www.lpi.usra.edu/meetings/lpsc2009/pdf/2420.pdf).
So the evaporation of water is slowed down at day time by the brine, not stopped.
But at night the brine may form again due to deliquescence. So we may get a day/night cycle of salt/brine from air humidity, without need for water supply from underground.

If there is any chance for water to stay liquid at or near the Martian surface, then via perchlorates.
Without perchlorate the water freezes at night and evaporates at day, with minor chance to ever stay liquid.

Obviously the chances are better closer to the poles than near the equator, because freezing (below -70°C) is less a problem than evaporation.

Edit: According to figure 7 of http://www.academia.edu/350635/Possible_physical_and_thermodynamical_evidence_for_liquid_water_at_the_Phoenix_landing_site the deliquescence threshold for Mg(ClO4)2 is near a relative humidity of 50% . At the same time the temperature needs to be above -70°C to form a brine.

Posted by: vikingmars Aug 18 2013, 12:09 PM

QUOTE (jmknapp @ Aug 15 2013, 05:57 PM) *
OK, here are the plots with selectable units:

WOW ! what a good work you did ! Thanks a lot Jmknapp ! wheel.gif wheel.gif wheel.gif wheel.gif wheel.gif

Posted by: PaulH51 Aug 19 2013, 03:03 AM

QUOTE (jmknapp @ Aug 18 2013, 07:39 AM) *
......quite a bit more is known about the Mars climate and of course there's the ground truth provided by Curiosity.

I also hope further experiments can be carried out with the new data provided by Curiosity. In the meantime I would like to thank Joe for providing the automated charts for Air Temperature and Pressure, especially useful as we can select the 'units' of measurement. I note however, that the source that he uses does not provide him with the data for the ground temperature.

Ground temperatures are now reported by Centro de Astrobiología (CSIC-INTA) on a new REMS page (as are air temperature and pressure etc.) I prefer Joe's pages as it tells a better story by showing the trends. However, if you want to see the historic records for minimum and maximum ground temperature, visit http://cab.inta-csic.es/rems/en/. Their data goes back to Sol 1, with only a small number of gaps up until the latest report on Sol 357. Updates to their page used to be daily, but recently this has been stretching out to weekly (this appears to be the same for all of the sites reporting REMS data)

I understand that 'detailed MSL REMS data' is deposited in the PDS archive and is available here via the http://an.rsl.wustl.edu/, but the last time I checked it only addressed the first 179 sols. The additional ground temperature details at the Spanish page may be of interest to some until more is added to the PDS archive.

I think the debate for the stability of surface liquids like brine or water will continue for some time, hopefully a detailed science paper that considers the detailed MSL data can finally put this debate to rest, one way or the other smile.gif

Posted by: jmknapp Aug 19 2013, 10:11 AM

I sent them an email to ask if they could add the ground temperature to the JSON/XML files. We'll see if that bears fruit. The maximum reported ground temperature appears to be up to 15°C warmer (or even more) than the max air temp on any given sol.

Thinking about the REMS IR sensor used to measure ground temperature brings up the question of what a sky IR temperature sensor might have shown, if they had included such. On Earth at least the sky IR temperature varies widely throughout the day depending on cloud cover and humidity, so it's a great way to show what's going on "up there." Maybe on Mars the picture would be relatively monotonous? Seems like the dust levels might have an effect.

The last REMS update on sol 357 was Curiosity's anniversary--maybe they're still partying?




Posted by: Gerald Aug 19 2013, 11:07 AM

QUOTE (PaulH51 @ Aug 19 2013, 05:03 AM) *
I understand that 'detailed MSL REMS data' is deposited in the PDS archive and is available here via the http://an.rsl.wustl.edu/, but the last time I checked it only addressed the first 179 sols.

Next release (Sols 180-269) is scheduled for Aug. 30, 2013: http://geo.pds.nasa.gov/missions/msl/index.htm.

Posted by: PaulH51 Aug 19 2013, 12:53 PM

QUOTE (Gerald @ Aug 19 2013, 07:07 PM) *
Next release (Sols 180-269) is scheduled for Aug. 30, 2013: http://geo.pds.nasa.gov/missions/msl/index.htm.

That additional data will please those with the ability to 'extract and interpret' the PDS REMS files smile.gif

I understand that this thread is is for Temperature and Pressure, but can anyone kindly point me to a thread for 'REMS wind'? TIA smile.gif

Posted by: Deimos Aug 19 2013, 02:11 PM

Re: the IR sky sensor: Spirit and Opportunity had that, and each got about a Mars year of good data. Of course they also had the bad habit of aiming it at the ground (MiniTES). Using IR for sky is very channel/wavelength dependent. Some probe 10s of meters, some probe through the atmosphere. So you'll see a range of brightness temperatures. But you can reconstruct the temperature profile of the bottom 2 km pretty well, and watch the diurnal formation of the boundary layer, and then the collapse and huge T inversion.

A big problem with a fix sensor is that the Sun is pretty likely to kill it if it can see the sky's signal.

Posted by: jmknapp Aug 19 2013, 02:47 PM

Thanks Deimos--didn't know that MiniTES did that.

Re the sun possibly damaging a fixed sensor, not sure about the type of sensor used on MSL (http://www.ipht-jena.de/fileadmin/user_upload/redaktion/pdf/Datenblaetter/TS-100-2011-10-28.pdf), but Melexis makes a nifty sensor (https://www.google.com/url?sa=t&rct=j&q=&esrc=s&source=web&cd=2&cad=rja&ved=0CDYQFjAB&url=http%3A%2F%2Fwww.melexis.com%2FAsset%2FIR-sensor-thermometer-MLX90614-Datasheet-DownloadLink-5152.aspx&ei=ay0SUpSwN6TN2AXd6oCoBg&usg=AFQjCNHtrsO3LD5RVtKakmtrz1UCPm-ayw&sig2=-yZ-L_cu6WTjGYa9InbxPg&bvm=bv.50768961,d.b2I) that boasts: "An optical filter (long-wave pass) that cuts off the visible and near infra-red radiant flux is integrated in the package to provide ambient and sunlight immunity... Special care is taken to cut off the visible light spectra as well as the NIR (near IR) before it reaches the sensitive sensor die."

Posted by: djellison Aug 19 2013, 06:10 PM

With MSL - the pointing problems mainly stem from frying ChemCam with too long a sun-point.

Posted by: jmknapp Aug 21 2013, 11:03 AM

Looks like they've resumed updating the REMS data stream--things are still following the general trend.

The PDS data, currently sols 0-179, is of course much more detailed, often with pressure and temperature records every second. With that kind of detail, temperature vs. pressure over a given sol can be plotted, tracing out a roughly closed loop (assuming that the following sol is similar). Here's a couple sols p-t data overlaid on a water phase diagram:



Sol 102 was the warmest sol so far (high temperature 9.4C/49F). Sol 172 had about the highest pressure. To date, all pressures have been above the triple point (black dot).

Posted by: Gerald Aug 23 2013, 04:44 PM

That's probably the most beautiful phase diagrams I ever saw, Joe! smile.gif

All we need is a cavern or something that prevents humidity from escaping and cooling down the ice by sublimation, before it can melt.

Posted by: jmknapp Aug 23 2013, 06:38 PM

Or, as Harder and you have mentioned, throw enough salt into the brine.

Saw an interesting claim that the atmospheric pressure being fairly close to the triple point is not just a coincidence. The argument goes that if conditions were such that liquid water could exist in significant quantities on the surface, said water would tend to fix carbon dioxide as carbonate. So the pressure (largely carbon dioxide) falls to the point where water no longer forms.

For a more detailed look at the first 180 sols in the PDS, I made this page:

http://curiosityrover.com/rems/pt.php

Note the sol selection box in the upper right. The sol can also be changed using the arrow keys--although that doesn't work with Firefox for some reason.

One feature I'd like to see explained: why does the pressure rise briefly near sunrise (6am lmst) as the temperature starts rising, but then reverse course to reach a low in the early afternoon?

Posted by: Deimos Aug 23 2013, 07:45 PM

I may mangle this, and I'll certainly oversimplify it, as the pressure variations are globally harmonic with multiple modes. But here goes: at 0500, the atmosphere is cold and dense everywhere around. Around sunrise, the symmetry is gone, and there are warming and cold areas. The atmosphere expands in the warming areas, both horizontally into adjacent cold areas and vertically. Thus, an "adjacent cold area" sees a rise. That is brief, as shortly after sunrise, the area becomes a warming area. As the boundary layer warms it expands. Vertical expansion leads to high pressure aloft and the net export of air out of the atmospheric column. As long as the high altitude pressure is elevated, air flows out of the high, and surface pressure falls. Shortly after the surface air starts to cool, the high becomes unsustainable. The cooling area becomes a net importer of air until the temperature has nearly stabilized. To recap: the rise occurs when adjacent areas are net exporters of air due to warming; the fall is when the local area is warming and a net exporter; the recovery occurs during cooling.

There are semidiurnal (twice a sol) modes originating higher in the atmosphere from dust absorption, and local effects from the crater & mound, so I may be over-interpreting that a bit. You can see some dust effects around sol 100.

Posted by: nprev Aug 24 2013, 03:12 PM

That rings true to me, Deimos. I was even more simplistically thinking that the air must be densest at the lowest temperature point of the day (pre-dawn) and expands when solar heating peaks in the early afternoon (allowing for thermal inertia at all times, of course).

The dust is surely a complicating factor in addition to local topography, though; not a simple system at all. Be interesting to see what happens during a global dust-storm.

Posted by: jmknapp Aug 24 2013, 05:11 PM

Yes, Deimos, thanks for simplifying it--can sort of get my head around that explanation. Maybe the wind information would flesh out the situation a little more along the lines you describe.

Posted by: Drkskywxlt Aug 30 2013, 04:06 PM

Yes, the repeatable pressure variations that REMS observes is primarily due to the diurnal and semidiurnal tides. The diurnal tide is responsible for the large-scale signal and the repeatable "shoulders" in the pressure data are mostly the semidiurnal tide. As Deimos said, complications come from dust loading and topographic effects in Gale.

Nprev...to answer your question about what would happen during a global dust storm: The Viking landers saw the pressure perturbation amplitudes (from the diurnal and semidiurnal tides) increase dramatically during the 1977 global dust storms, but they were at fairly high latitudes. For MSL, because of its tropical location, the diurnal amplitude would probably decrease to nearly zero, but the semidiurnal amplitude would likely increase a fair bit.

Posted by: jmknapp May 12 2014, 01:43 AM

FYI, the SETI Institute has a talk scheduled for 12:00pm Pacific on Tuesday May 13th:

http://www.seti.org/weeky-lecture/carbon-dioxide-snowfalls-polar-caps-and-climate-mars

Posted by: jmknapp Sep 12 2014, 02:27 AM

FWIW, the high temperature for the last few sols has been flirting with or above freezing:

http://curiosityrover.com/rems/

Posted by: serpens Sep 12 2014, 02:57 AM

Those anomalous dips in both max and min temperatures for periods around around sol 292 and sol 500 have got to draw attention. MSL was stationary over the period sol 494 to 503 and so there should be no external or movement related reason for that drop. The correlation between max and min deviations is pretty loose. Is this sensor unreliability or something else? The temptation is of course to ignore these outliers and admire the smoothly plotted curves but we learned a bitter lesson on that approach with respect to the Antarctic ozone layer attenuation.

Posted by: nprev Sep 12 2014, 03:29 AM

The data could've been corrupted in a variety of ways aloft or on the ground. I'd say it's safe to ignore the extreme outliers.

I wonder what the actual ground temperatures are since the meteorological instrumentation is a meter or more above that. Seem to recall reading that temps right at the surface might be some ridiculously disparate number like 50 deg F or so greater than that of the air in the daytime.

Posted by: Vultur Sep 12 2014, 05:23 AM

QUOTE (jmknapp @ Sep 12 2014, 03:27 AM) *
http://curiosityrover.com/rems/



The difference between the daily high and low temperatures is huge! Looks like almost 80 degrees Celsius difference...

(And why does the air pressure vary so much over the seasons? Is that because of CO2 freezing out of the atmosphere onto the polar cap in winter?)

Posted by: Gerald Sep 12 2014, 09:25 AM

QUOTE (nprev @ Sep 12 2014, 05:29 AM) *
I wonder what the actual ground temperatures are since the meteorological instrumentation is a meter or more above that.

Besides air temperature sensors, REMS also has three thermopiles for remote measurement of the ground temperature. But just thermopile A returns data with a reasonable noise level (as far as I've seen). Data of thermopiles B and C haven't been cleaned of all systematic measurement errors. But they've made good progress with the calibration of the thermopile A data. Warm-up, shadowing, and maybe other effects needed to be considered. Since one of the two booms has been damaged, there isn't as much redundancy left as originally planned. See the posts about PDS REMS data, starting http://www.unmannedspaceflight.com/index.php?s=&showtopic=7613&view=findpost&p=199252.

QUOTE (Vultur @ Sep 12 2014, 07:23 AM) *
And why does the air pressure vary so much over the seasons? Is that because of CO2 freezing out of the atmosphere onto the polar cap in winter?

Yes, mainly. http://www.msss.com/http/ps/seasons/seasons.html, http://faculty.washington.edu/sew2/publications/Wood-Paige-1992-Mars-co2-cycle-fit-viking-pressure.pdf. Maybe, some regional or local effect is superposed.


Posted by: jmknapp Sep 12 2014, 09:31 AM

The link to ground temperature on that http://curiosityrover.com/rems/ will show plots from the first 180 sols. Looking at sol 170, the high air temp was about 6°C while the high ground temp (brightness temp) was 0°C. That's interesting. I suppose on Earth it's the same--on a hot day the ground will be cooler. Maybe it's a factor of how brightness temperature is defined though, or a calibration issue. I need to get updated PDS data on that page for sols in the winter. Maybe it's only on cold days that the ground temp is far above the air temp?

The extreme outliers are easy to dismiss as anomalous (although current Mars weather would become infinitely more interesting if they were real) but I find the smaller blips more intriguing. Like at around sol 680, did the high temperature really drop around 10-15°C for a week or so? Ten degrees here, ten degrees there, pretty soon you're talking real weather.

Posted by: Gerald Sep 12 2014, 09:47 AM

Near the boundary of high- and low-thermal-inertia, e.g. rock and sand, a change in the wind direction may easily change the air temperature. Same near shadows or inclined terrain with varying sun exposure. Another effect may be the relation between RTG and wind. Or changes in voltage during arm, camera, or SAM activities, which may affect REMS data. Data for low temperatures tend to be noisier than data for higher temperatures.
Proper interpretation of the data and modeling won't be easy.

Posted by: jmknapp Sep 12 2014, 10:15 AM

Ah, those are good points. Around sol 678 here's the position of the rover:

http://curiosityrover.com/tracking/drivetrack.php?drivenum=209

...in a sandy depression. So maybe it's less weather and more finding a cool spot, so to speak.

Posted by: jmknapp Sep 17 2014, 12:15 PM

Pressure throughout the day over the first ~540 sols:



That's not quite a full Mars year, but all that's in the PDS so far. The seasons are for the northern hemisphere.

Posted by: Gerald Sep 17 2014, 01:03 PM

Wow, what a lot of work, scanning through all these files!
Interesting, that there is a peak in the morning near 8 a.m.
Maybe a superposition of harmonics of the diurnal tide. Or might be related to slope winds.

Posted by: jmknapp Sep 17 2014, 05:27 PM

And the peak is higher in the northern hemisphere fall than the spring--consistent with the north polar cap giving up its CO2 and H2O to the atmosphere, more than the south polar cap? On the other hand, MSL is some 60m higher than at the start of the mission, so maybe that accounts for some of the difference.

Here's a similar chart for temperature:


Posted by: Gerald Sep 17 2014, 08:43 PM

Due to the http://en.wikipedia.org/wiki/Orbital_eccentricity of http://en.wikipedia.org/wiki/Mars#Orbit_and_rotation around the Sun, and the larger distance in southern winter, we've more CO2 freeze-out in the southern winter than in the northern.
This will account for most of the annual pressure oscillation, and the higher amplitude between southern winter and southerm spring.

The pressure difference due to an altitude difference of 60 m should be roughly 0.55%, or 0.05 mbar:
According to the http://en.wikipedia.org/wiki/Barometric_formula the decrease of the pressure with height is roughly proportional to the gravitational acceleration, the molar mass of the air, and to the height difference, inversely proportional to the temperature, for small height differences.
With a gravitational acceleration of 0.376 g, a molar mass of 12 + 2*16 = 44 for CO2 on Mars and 0.8 * 2 * 14 + 0.2 * 2 * 16 = 28.8 for 80% N2 and 20% O2 on Earth, meaning a factor of 1.53 for Mars relative to Earth, 210 K mean surface temperature for Mars and 288 K for Earth, we get a factor of 1.37 for the inverted temperature of Mars relative to Earth, together 0.376 * 1.53 * 1.37 = 0.788 times the value on Earth for the relative decrease of pressure for the same (small) height difference. Or divide the http://en.wikipedia.org/wiki/Scale_height of Earth (8.5 km) by the scale height of Mars (11.1 km), 8.5 / 11.1 = 0.766 as a cross-check.

The http://en.wikipedia.org/wiki/Atmospheric_pressure on Earth on sea level varies 1.2 kPa for every 100 m, meaning 1.2 kPa / 101.32 kPa = 1.18%, hence for Mars and 60 m height difference 1.18% * 0.60 * 0.78 = 0.55%.
0.55% of 8.5 mbar are about 0.047 mbar. That's about one color step in your graphics.

Posted by: ngunn Sep 17 2014, 10:03 PM

QUOTE (Gerald @ Sep 17 2014, 09:43 PM) *
According to the http://en.wikipedia.org/wiki/Barometric_formula the decrease of the pressure with height is roughly proportional to the gravitational acceleration, the molar mass of the air, and to the height difference, inversely proportional to the temperature, for small height differences.
With a gravitational acceleration of 0.376 g, a molar mass of 12 + 2*16 = 44 for CO2 on Mars and 0.8 * 2 * 14 + 0.2 * 2 * 16 = 28.8 for 80% N2 and 20% O2 on Earth, meaning a factor of 1.53 for Mars relative to Earth, 210 K mean surface temperature for Mars and 288 K for Earth, we get a factor of 1.37 for the inverted temperature of Mars relative to Earth, together 0.376 * 1.53 * 1.37 = 0.788 times the value on Earth for the relative decrease of pressure for the same (small) height difference. Or divide the http://en.wikipedia.org/wiki/Scale_height of Earth (8.5 km) by the scale height of Mars (11.1 km), 8.5 / 11.1 = 0.766 as a cross-check.



This takes me back. One of the first things I posted here was this hand drawn diagram comparing four atmospheres. (The topic didn't catch on - it's a one post thread!)


QUOTE (ngunn @ May 22 2006, 03:53 PM) *
Thought I'd try starting a new place for this as our discussions on the subject so far have been popping up all over the place. I find them hard to locate afterwards, and it must be even worse for newcomers trying to follow the topic.

To kick off here is the diagram I use for comparing the atmospheres of Titan and the terrestrial planets:
http://www.unmannedspaceflight.com/index.php?act=attach&type=post&id=5799

Posted by: Vultur Sep 21 2014, 05:36 AM

QUOTE (Gerald @ Sep 12 2014, 09:25 AM) *
Yes, mainly. http://www.msss.com/http/ps/seasons/seasons.html, http://faculty.washington.edu/sew2/publications/Wood-Paige-1992-Mars-co2-cycle-fit-viking-pressure.pdf. Maybe, some regional or local effect is superposed.


Ah, cool, thanks.

Posted by: HSchirmer May 12 2017, 09:57 PM

Agreed,

QUOTE (atomoid @ May 11 2017, 10:03 PM) *
at near zero humidity and pressure over geologic timescales


Well, not entirely zero-humidity.
Low absolute, but IIRC the rover has seen 75% or higher relative humidity.


Posted by: serpens May 14 2017, 12:12 AM

[quote name='HSchirmer' date='May 12 2017, 09:57 PM' post='235777']
Agreed, Well, not entirely zero-humidity. Low absolute, but IIRC the rover has seen 75% or higher relative humidity.......
.....Our results suggest that water vapor can be stored within the regolith via deliquescence of calcium perchlorate.....
....Therefore, this process may play an important role in the near-surface water cycle on present-day Mars.
quote]

With atmospheric water content varying between some 10 to 60 ppm at Gale Cater it is difficult to see any significant effect in this locality.

Posted by: HSchirmer May 15 2017, 02:41 AM

QUOTE (serpens)
QUOTE (HSchirmer)

Agreed, Well, not entirely zero-humidity. Low absolute, but IIRC the rover has seen 75% or higher relative humidity.......
.....Our results suggest that water vapor can be stored within the regolith via deliquescence of calcium perchlorate.....
....Therefore, this process may play an important role in the near-surface water cycle on present-day Mars.


With atmospheric water content varying between some 10 to 60 ppm at Gale Cater it is difficult to see any significant effect in this locality.


QUOTE
https://www.hou.usra.edu/meetings/lpsc2017/pdf/2972.pdf
Mars Science Laboratory.
...
Our results suggest that water vapor can be stored within the regolith via deliquescence of calcium perchlorate.
In the top, few meters of soil, the stored water vapor would be released by sublimation after the solution transitions
to the ice phase.
However, in the deeper subsurface, the attenuated temperature and resulting relative humidity,
avoid the transition to the ice phase. In this scenario, a solution is present for several hours...


Eh, liquid water on mars, for several hours each day.

Posted by: Gerald May 15 2017, 04:32 PM

QUOTE (HSchirmer @ May 15 2017, 03:41 AM) *
...Eh, liquid water on mars, for several hours each day.

We discussed these possibilities a few years ago in http://www.unmannedspaceflight.com/index.php?s=&showtopic=7489&view=findpost&p=192435, see http://www.unmannedspaceflight.com/index.php?s=&showtopic=7489&view=findpost&p=192692, or the posts starting http://www.unmannedspaceflight.com/index.php?s=&showtopic=7489&view=findpost&p=202487.
I'd suggest to continue the discussion about the possibility of liquid water in Gale, including brines, with new results getting available, in the dedicated thread.

Posted by: serpens May 16 2017, 02:37 AM

Indeed. Perhaps one of our overworked moderators could transfer the relevant posts?
There is an hypothesis that brines could form through deliquescence. However there are a number of empirical findings that mitigate against this. First is the presence of kieserite which indicates absolute desiccation. Secondly the Thermal and Electrical Conductivity Probe (TECP) deployed by the Phoenix lander did not detect any indication of water vapour or liquid in the regolith despite a much more likely environment than Gale. Thirdly no indications of frost have been identified by Curiosity although to be fair any such would be limited to a micron or so. Finally it is absolute humidity that is the important measure for atmospheric water. The actual amount of water available rather than relative humidity which is simply the water vapour measured as a percentage of what the atmosphere could hold at a specific temperature. We need an adjective more intense than nanoscopic to describe 10 to 60 ppm.

Posted by: HSchirmer May 17 2017, 02:05 AM

QUOTE (serpens @ May 16 2017, 02:37 AM) *
First is the presence of kieserite which indicates absolute desiccation.


Ok, interesting point, but,
isn't the presence of MgSO4*H2O, i.e. a hydrated magnesium sulfate, evidence of some water?
Wouldn't MgSO4 without any H20 at all be absolute dessication?

Posted by: serpens May 17 2017, 04:38 AM

On Mars, kieserite is the end state dehydration product for the MgSO4 hydrates. Water is tightly bound to the crystal structure in monohydrates and it would take temperatures over 300 C to force the water out of kieserite. However kieserite is unstable with respect to hydration and is very sensitive to humidity. Unless there is something we don't understand about the stability field of kieserite in the Martian environment then the presence of kieserite implies absolute dessication of the environment (not of the kieserite).

Posted by: Gerald May 17 2017, 12:54 PM

When comparing the stability diagram of Mg(ClO4)2 brine in figure 2 of http://comp.uark.edu/~vchevrie/sub/papers/1305%20-%20Gough%20-%202011%20-%20EPSL%20-%20Perchlorates%20deliquescence.pdf with the stability diagram of kieserite in figure 3 of https://ntrs.nasa.gov/archive/nasa/casi.ntrs.nasa.gov/20050174605.pdf, I presume the possibiliy of an Mg(ClO4)2 brine in the presence of kieserite. It would form above -50°C at a relative humidity near 50%, and stay metastable down to a relative humidity of 20%. For temperature above -50°C, kieserite is stable according to the first paper. However, according to other papers, e.g. http://www.getty.edu/conservation/our_projects/science/salt/balboni2010.pdf, kieserite may transform less easily into hexahydrate and epsomite, such that we may lower the temperature further, where a Mg(ClO4)2 brine can form easier due to the higher relative humidity at low temperatures, but only weak dependency of the lower bound of relative humidity for brine formation on temperature.

Posted by: HSchirmer May 17 2017, 08:27 PM

QUOTE (serpens @ May 17 2017, 05:38 AM) *
However kieserite is unstable with respect to hydration and is very sensitive to humidity.
Unless there is something we don't understand about the stability field of kieserite in the Martian environment
then the presence of kieserite implies absolute dessication of the environment (not of the kieserite).


QUOTE
http://www.indiana.edu/~geosci/bish/Vaniman%20et%20al%20sulfates%20on%20Mars.pdf
Under colder conditions on Mars (220 K equatorial average) and at typical Mars surface pressure
(5 torr) and diurnal RH ranging from , 1% to 100% (summer equatorial average 50% RH),
...
experiments in the environmental cell show that [kieserite] is easily hydrated on exposure to elevated humidity
(55% RH) where it converts to hexahydrite and then epsomite.


Eh, perhaps this area hasn't see 55% RH recently.
Or, perhaps re-hydration doesn't happen, because it is thermodynamically favorable, but kinetically slow.

QUOTE (http://www.planetary.brown.edu/pdfs/3913.pdf)
Vaniman et al. [2006] have shown that under Martian temperature conditions (constant 243 K trials)
but ~1 bar pressure, kieserite hydrates to hexahydrite in ~1000 h at 100% RH.


So, kiersite proves that this site hasn't seen a biblical flood, underwater for 40 days and 40 nights (~1000 hours), anytime recently.

QUOTE
At 100% RH in lower temperatures (193 K), the water vapor does not hydrate kieserite,
but accumulates as surface frost on the mineral [Vaniman et al., 2006].


So, adding water to kiersite for a few hours per day might not result in hydrates, only a wet rock.
The twice daily temperature swings, ice-liquid-vapor, might effectively limit the time that liquid water is chemically available,
which would effectively prevent rehydration.

PS

http://meteorites.wustl.edu/abstracts/lpsc37/a_l06w01.pdf

In a separate study of phase transformations among the hydrated Mg sulfates [7],
we found no evidence of any rehydration of the kieserite (MgSO4·H2O) at 30% RH
and 50ºC within the first 24 hours.

Posted by: ngunn May 17 2017, 11:39 PM

I just want to say thank you to all involved for having this informative discussion here. It's exactly the sort of thing (besides the excellent image work) that I come here for.

Posted by: serpens May 18 2017, 12:58 AM

Hydration of kieserite under Martian conditions would indeed be kinetically slow. Sluggish is the term used in some experimental results. However we are considering extraordinarily long timescales. If the kieserite was formed as a primary evaporate then the current slow erosion rate would have resulted in long exposure times at or near the surface. 75% humidity has been recorded at Gale and the thing is that under current Martian conditions, over Martian surface timescales kieserite could rehydrate, possibly to an amorphous state provided that there was a sufficiently high absolute humidity. The most likely end product would be starkeyite which is stable in this environment. But 7 to 9.6 mbar and 100% RH with 10 to 60 ppm water hardly meets the definition of a biblical flood and if all water in the atmospheric column precipitated there would be just a few microns on the surface.

Posted by: HSchirmer May 18 2017, 01:49 AM

QUOTE
Hydration of kieserite under Martian conditions would indeed be kinetically slow.
...
However we are considering extraordinarily long timescales.
...
If the kieserite was formed as a primary evaporate then the current slow erosion rate
would have resulted in long exposure times at or near the surface.


That part is where it gets interesting.
Exposure at or near the surface could be what keeps it dry.

I think of crossing Mar\'s dunes as similar to humans walking across hot sand at the beach.
Think of the microclimate of desert sands, superheated air and shimmering mirages.

With a thin atmosphere, convection will be much less efficient at cooling.
With fluffy aeolian dust, conduction will be much less efficient.
Of course any surface soaking up the sun all day is going to be (relatively) \"hot\" and \"bone dry\".

So, yes, there are salts with long exposure times, literally geologic timescales...

QUOTE (Experimental stability of magnesium sulfate hydrates that may be present on Mars)
http://\"http://www.sciencedirect.com/science/article/pii/S0016703706020965\"
Several of the hydrates also show significant metastable extensions, such that phase boundaries can only be approximated. For example, kieserite, which has been reported on Mars from OMEGA data, in addition to having a distinct stability region,
is resistant to transformation and persists throughout temperature-RH space until very high relative humidities are achieved.


But the daily variations in surface temperature and RH seem to ensures that the visible surface
constantly over or undershoots the combination of temperature and RH needed for hydration reactions at the visible surface.

You\'d probably get water adsorbed onto the salt, but it wouldn\'t be there long enough to incorporate.
There\'s plenty of time, but not enough time where the right combinations for weathering occur simultaneously.

EDIT-
Ah, that may be a threshold issue-
MgSO4-H20 +6H2O => MgSO4-7H2O is the thermodynamically favored reaction. (at least at Earth pressure and temp).
So, it requires about 70% by weight of water to form the next stable hydrate.
If you don\'t reach 70% weight by water, then hydration may be reversible.

QUOTE (Experimental studies of the mechanism and kinetics of hydration reactions- Energy Procedia 48 ( 2014 ) 394 – 404)
http://www.sciencedirect.com/science/article/pii/S1876610214003087/pdf?md5=3742b3c58a08d9fb8b17175a816e7fd8&pid=1-s2.0-S1876610214003087-main.pdf

though not yet fully understood, it appears plausible that diffusive water transport across
a barrier product layer formed at the reaction interface may often be the rate limiting step.

The water uptake curves show that below the DRH of MgSO4∙H2O (red curves) a partial hydration and formation
of MgSO4∙6H2O takes place. The thermodynamically stable hydration product under these conditions is MgSO4∙7H2O.
Full hydration to MgSO4∙6H2O was not achieved within 60 hours (only the first 25 hours are shown in Fig. 2).
Above the DRH of MgSO4∙H2O the hydration rate is significantly increased and the formation of MgSO4∙6H2O is complete within 25 hours (blue curves).


So, kieserite may adsorb water molecules into a non-reactive film, which blocks anything from happening until there is an actual liquid phase. Think of the protective patina that forms on copper; or even better, aluminum, which is, paradoxically so-reactive with oxygen that it instantly forms a non-reactive patina... http://news.stanford.edu/pr/00/aluminum511.html

However, dig down into a dune, and should be cooler, and more humid, and much more interesting...

Posted by: serpens May 18 2017, 04:06 AM

Just as an aside, you may like to consider the interactions between the hypothesised brine, kieserite and other hydrated sulphates.

Posted by: Gerald May 18 2017, 09:16 AM

Pure H2SO4 is so hygroscopic, that it pulls the water out of dry sugar, leaving carbon.
Although perchlorates arent't quite as hygroscopic, brines near https://en.wikipedia.org/wiki/Eutectic_system may still bind water molecules strong enough, that they aren't available for reaction with kieserite. I didn't find information whether in such a settings, a sulfate-perchlorate https://en.wikipedia.org/wiki/Double_salt would form, or whether double salts like Mg(SO4)(ClO4)·nH2O even exist.

We can find out, whether films of brines are possible in Gale. But how can we learn, whether they are actually there?

Posted by: serpens May 19 2017, 12:27 AM

Analysis of Phoenix WLC results led to the conclusion that there was between 0.4 to 0.6 % by weight of perchlorate in the regolith with SAM indicating a similar outcome for Rocknest. With only 10 to 60 ppm atmospheric water content at Gale and low RH I suspect that any brine that did form would consist of really minuscule, isolated droplets rather than extended films or pools of liquid.

While the detection of ephemeral hydrated salts in some recent RSL has been nominated as proof of brines by some, I question whether the phase change observed is not a function of the exposure of previously protected hydrated salts to the atmosphere as a result of a dry slip. Such an effect was recorded by Spirit at Tyrone. Regardless I suspect that there will need to be boots on the ground before the question of brines can be resolved.

Posted by: HSchirmer May 19 2017, 01:03 AM

QUOTE (serpens @ May 19 2017, 12:27 AM) *
I suspect that any brine that did form would consist of really minuscule,
isolated droplets rather than extended films or pools of liquid.


Agreed that there won't be pools, but I'm actually leaning toward chemically active films, rather than solution droplets.

Many (many) years ago, I prepared divalent cation solutions and spectrophotometer baselines for enzyme binding spectroflouroscopy. (I.e. I mixed chemicals and did precise measurements of how color changes with concentration).
Part of that included reviewing how water works. How water as a polar fluid shields charges, how water structure changes when things are dissolved. Basically, treating water as a jenga piece, how do you remove it, or how do you add it?

This is relevant here, because the activated adsorbed single molecule films that salts accumulate, are going to be reactive.
Oddly, due to stoichiametric requirement, some reactions just won't work unless you get eh, 6 water molecules,
while other reactions will work with just 1, 2, 3 or more..



QUOTE (serpens @ May 19 2017, 12:27 AM) *
I suspect that there will need to be boots on the ground before the question of brines can be resolved.


Agreed.

Powered by Invision Power Board (http://www.invisionboard.com)
© Invision Power Services (http://www.invisionpower.com)